Must Be True Questions - - Question 12
Governments have only one response to public criticism of socially necessary services: regulation of the activity of ...
Replies
crichburg1 January 5, 2019
I agree with Rebecca
Ravi January 5, 2019
@Rebecca-Alvarado and @ crichburg1,Great questions! Diagramming on this question isn't necessary, but you
may find it helpful. The stimulus contains conditional logic, and we
can break down the contents of the passage into the following diagram:
Public criticism of service - > regulation - > increased cost
Child care is publicly criticized - > there will be regulation
The question stem asks us to select the answer choice that can be
inferred based on the contents of the passage. Given the diagram we
have, we can look at the answers and see if they match up with
anything we have in it.
Answer A is incorrect because we are given no information about
whether child care will improve. While regulation is intended to
improve the quality of something, this simply isn't a guarantee, so it
cannot be inferred. We can get rid of this choice.
Answer B says the cost of providing child-care services will increase.
This is correct because we know that regulation leads to an increase
in cost (making the activity more expensive), and we know that there
will be regulation because the stimulus says that the government is
certain to respond. This is our answer choice, as it is an inference
that we can make.
Answer C is incorrect because there is no information in the passage
dealing with the government funding advances in child care. We can
easily eliminate this answer.
Answer D is incorrect because we do not know that government always
responds to public criticism; all we know is that the government has
only one response to public criticism. Whether or not they always
respond is another question that we don't have the answer to. Note,
however, that we do know the government is certain to respond to
public criticism of child care services because the last sentence of
the stimulus tells us. However, as mentioned, this statement is far
too broad, and we can eliminate it.
Answer E is incorrect because it uses false logic in its statement.
The stimulus tells us that regulation - ->increased cost, and E is
saying not regulation - ->no increase in cost. This is the equivalent
of translating A - ->B into /A - ->/B. This is an incorrect
contrapositive for A - ->B; the correct contrapositive for A - ->B is
/B - ->/A. Thus, this answer is wrong and we can eliminate it.
Does this make sense? Let us know if you have more questions!
Ravi January 5, 2019
@crichburg1 tagging you here because the tag in my answer above didn't workGabriela-Diaz April 4, 2020
Hi I do not understand why B is correct.The first sentence notes governments have only one response to public criticism of socially necessary services: regulation of the activity of providing those services.
And goes on to say regulation makes it more expensive.
B says it child care will be more expensive. But doesn't that rely on the assumption that they will respond and enact those regulations just because public criticism happend of a socially necessary service?
D seems to address that assumption so that is why I picked it.